|
9 | 9 | \large
|
10 | 10 | \setlength{\baselineskip}{1.2em}
|
11 | 11 | \ifpreface
|
12 |
| - \input{../../../global/preface} |
13 |
| -\newgeometry{left=2cm,right=2cm,top=2cm,bottom=2cm} |
| 12 | + \input{../../../global/preface} |
| 13 | + \newgeometry{left=2cm,right=2cm,top=2cm,bottom=2cm} |
14 | 14 | \else
|
15 |
| -\newgeometry{left=2cm,right=2cm,top=2cm,bottom=2cm} |
16 |
| -\maketitle |
| 15 | + \newgeometry{left=2cm,right=2cm,top=2cm,bottom=2cm} |
| 16 | + \maketitle |
17 | 17 | \fi
|
18 | 18 | %from_here_to_type
|
19 | 19 | \begin{problem}
|
20 |
| - Assume \(V \subset \mathbbm{A}_k^{n}\) is irreducible, and \(p \in V\). |
21 |
| - Let \(m_p := \{f \in k[V] : f(p) = 0\}, M_p := \{f \in k[x_1,\cdots,x_n]: f(p) = 0\}\). |
22 |
| - Prove that \(m_p \cong M_p/\ide(V)\). |
| 20 | + Assume \(V \subset \mathbbm{A}_k^{n}\) is irreducible, and \(p \in V\). |
| 21 | + Let \(m_p := \{f \in k[V] : f(p) = 0\}, M_p := \{f \in k[x_1,\cdots,x_n]: f(p) = 0\}\). |
| 22 | + Prove that \(m_p \cong M_p/\ide(V)\). |
23 | 23 | \end{problem}
|
24 |
| - |
25 | 24 | \begin{solution}
|
26 | 25 | Consider \( \theta : M_p \to m_p ,\theta(f) := f + \ide(V)\).
|
27 |
| - Since \( p \in V\) we get \(\theta\) is well-defined. |
28 |
| - And easily we get \(\theta\) is homomorphism. Now consider \(\ker \theta\). |
29 |
| - Obviously \(\ide(V) \subset \ker \theta\), now we prove \(\ker \theta \subset \ide(V)\). |
30 |
| - Assume \( f \in \ker\theta\), to prove \(f \in \ide(V)\). |
31 |
| - Since \(\theta(f)=\ide(V)\), we get \(f+\ide(V)=\ide(V)\), so \(f \in \ide(V)\). |
32 |
| - So \(\ker \theta = \ide(V)\). And easily \(\theta\) is surjective, so we get |
33 |
| - \(m_p=M_p/\ker \theta=M_p / \ide(V)\). |
| 26 | + Since \( p \in V\) we get \(\theta\) is well-defined. |
| 27 | + And easily we get \(\theta\) is homomorphism. Now consider \(\ker \theta\). |
| 28 | + Obviously \(\ide(V) \subset \ker \theta\), now we prove \(\ker \theta \subset \ide(V)\). |
| 29 | + Assume \( f \in \ker\theta\), to prove \(f \in \ide(V)\). |
| 30 | + Since \(\theta(f)=\ide(V)\), we get \(f+\ide(V)=\ide(V)\), so \(f \in \ide(V)\). |
| 31 | + So \(\ker \theta = \ide(V)\). And easily \(\theta\) is surjective, so we get |
| 32 | + \(m_p=M_p/\ker \theta=M_p / \ide(V)\). |
34 | 33 | \end{solution}
|
35 | 34 | \begin{problem}
|
36 |
| - Prove that \(M_p/M_p^2+\ide(V) \cong m_p/m_p^2\). |
| 35 | + Prove that \(M_p/M_p^2+\ide(V) \cong m_p/m_p^2\). |
37 | 36 | \end{problem}
|
38 |
| - |
39 | 37 | \begin{solution}
|
40 |
| - Consider \(\theta : M_p \to m_p/m_p^2, f \mapsto f|_V+m_p^2\). Easily \(\theta\) is homomorphism and surjective. |
41 |
| - Now we prove \(\ker \theta = M_p^2+\ide(V)\). |
| 38 | + Consider \(\theta : M_p \to m_p/m_p^2, f \mapsto f|_V+m_p^2\). Easily \(\theta\) is homomorphism and surjective. |
| 39 | + Now we prove \(\ker \theta = M_p^2+\ide(V)\). |
42 | 40 |
|
43 | 41 | On one hand, assume \(f \in \ker \theta\), i.e., \(f|_V \in m_p^2\).
|
44 |
| - Then \(\exists g_1,\cdots,g_n,h_1,\cdots,h_n \in m_p\) such that \(f|_V=\sum_{k=1}^{n} g_k h_k\). |
45 |
| - Assume \(g_k=G_k|_V,h_k=H_k|_V\). Then \(G_k,H_k \in M_p\). |
46 |
| - Consider \(f-\sum_{k=1}^{n} G_k H_k=:h \in k[x_1,\cdots,x_n]\), easily to know \(h(x)=0,\forall x \in V\). |
47 |
| - So \(h \in \ide(V)\), thus \(f \in M_p^2+\ide(V)\). |
| 42 | + Then \(\exists g_1,\cdots,g_n,h_1,\cdots,h_n \in m_p\) such that \(f|_V=\sum_{k=1}^{n} g_k h_k\). |
| 43 | + Assume \(g_k=G_k|_V,h_k=H_k|_V\). Then \(G_k,H_k \in M_p\). |
| 44 | + Consider \(f-\sum_{k=1}^{n} G_k H_k=:h \in k[x_1,\cdots,x_n]\), easily to know \(h(x)=0,\forall x \in V\). |
| 45 | + So \(h \in \ide(V)\), thus \(f \in M_p^2+\ide(V)\). |
48 | 46 |
|
49 |
| - On the other hand, assume \(f \in M_p^2+\ide(V)\), to prove \(\theta f =0\). |
50 |
| - Assume \(f=\sum_{k=1}^{n} G_k H_k+h\), where \(G_k,H_k \in M_p\) and \(h \in \ide(V)\). |
51 |
| - Then \(\theta f = \sum_{k=1}^{n} g_k h_k +m_p^2\), where \(g_k=G_k|_V,h_k=H_k|_V\). |
52 |
| - So we get \(\theta f = m_p^2=0\). |
| 47 | + On the other hand, assume \(f \in M_p^2+\ide(V)\), to prove \(\theta f =0\). |
| 48 | + Assume \(f=\sum_{k=1}^{n} G_k H_k+h\), where \(G_k,H_k \in M_p\) and \(h \in \ide(V)\). |
| 49 | + Then \(\theta f = \sum_{k=1}^{n} g_k h_k +m_p^2\), where \(g_k=G_k|_V,h_k=H_k|_V\). |
| 50 | + So we get \(\theta f = m_p^2=0\). |
53 | 51 |
|
54 |
| - Finally we get \(m_p/m_p^2 = M_p/ \ker \theta = M_p / M_p^2+\ide(V)\). |
| 52 | + Finally we get \(m_p/m_p^2 = M_p/ \ker \theta = M_p / M_p^2+\ide(V)\). |
55 | 53 | \end{solution}
|
56 | 54 | \begin{problem}
|
57 |
| - Assume \(V \subset \mathbbm{A}_k^n\) is irreducible, and \(p \in V,f \in k[V], f(p)\neq 0\). |
58 |
| - Consider \(V_f:= \{x \in V: f(x)\neq 0 \}\). Let \(\theta: V_f \to \mathbbm{A}_k^{n+1},x \mapsto (x,\frac{1}{f(x)})\). |
59 |
| - Let \(U = \theta(V_f)\), prove that \(T_p V \cong T_{\theta(p)} U\). |
| 55 | + Assume \(V \subset \mathbbm{A}_k^n\) is irreducible, and \(p \in V,f \in k[V], f(p)\neq 0\). |
| 56 | + Consider \(V_f:= \{x \in V: f(x)\neq 0 \}\). Let \(\theta: V_f \to \mathbbm{A}_k^{n+1},x \mapsto (x,\frac{1}{f(x)})\). |
| 57 | + Let \(U = \theta(V_f)\), prove that \(T_p V \cong T_{\theta(p)} U\). |
60 | 58 | \end{problem}
|
61 | 59 |
|
62 | 60 | \begin{solution}
|
63 |
| - Write \(k[\mathbbm{A}_k^n]=k[x_1,\cdots , x_n ,y]\). Assume \(V = \van(I)= \van(f_1,\cdots, f_m)\), where \(f_i \in k[x_1,\cdots,x_n ]\). |
64 |
| - Then \(U = \van(f_1,\cdots,f_n, yf-1)\). Now consider \(\tau: T_p(V) \to \mathbbm{A}_k^{n+1}, \tau(x):=(x,\frac{1}{f(p)}-\frac{f_p^{(1)}(x)}{f^2(p)})\). |
65 |
| - Now we prove \(\tau(T_p V)=T_{\theta(p)} U\). |
66 |
| - Only need to prove \((yf-1)_{\theta(p)}^{(1)}(\tau(x))=0\). |
67 |
| - i.e., \(\frac{f_p^{(1)}(x)}{f(p)}+f(p)(y-\frac{1}{f(p)})=0\), where \(y = \frac{1}{f(p)}-\frac{f_p^{(1)}}{f^2(p)}\). |
68 |
| - Substitute \(y\) into the equation, we get is't obvious. |
| 61 | + Write \(k[\mathbbm{A}_k^n]=k[x_1,\cdots , x_n ,y]\). Assume \(V = \van(I)= \van(f_1,\cdots, f_m)\), where \(f_i \in k[x_1,\cdots,x_n ]\). |
| 62 | + Then \(U = \van(f_1,\cdots,f_n, yf-1)\). Now consider \(\tau: T_p(V) \to \mathbbm{A}_k^{n+1}, \tau(x):=(x,\frac{1}{f(p)}-\frac{f_p^{(1)}(x)}{f^2(p)})\). |
| 63 | + Now we prove \(\tau(T_p V)=T_{\theta(p)} U\). |
| 64 | + Only need to prove \((yf-1)_{\theta(p)}^{(1)}(\tau(x))=0\). |
| 65 | + i.e., \(\frac{f_p^{(1)}(x)}{f(p)}+f(p)(y-\frac{1}{f(p)})=0\), where \(y = \frac{1}{f(p)}-\frac{f_p^{(1)}}{f^2(p)}\). |
| 66 | + Substitute \(y\) into the equation, we get is't obvious. |
69 | 67 |
|
70 |
| - Obviously \(\tau\) is injective, so it's isomorphic. |
| 68 | + Obviously \(\tau\) is injective, so it's isomorphic. |
71 | 69 | \end{solution}
|
72 |
| - |
| 70 | +\begin{figure} |
| 71 | + \centering |
| 72 | + \includegraphics{~/Pictures/pic2/365/IMG_3135.JPG} |
| 73 | +\end{figure} |
73 | 74 | \end{document}
|
0 commit comments